LSAT and Law School Admissions Forum

Get expert LSAT preparation and law school admissions advice from PowerScore Test Preparation.

 Administrator
PowerScore Staff
  • PowerScore Staff
  • Posts: 8917
  • Joined: Feb 02, 2011
|
#25921
Complete Question Explanation
(The complete setup for this game can be found here: lsat/viewtopic.php?t=5457)

The correct answer choice is (C)

The question stem establishes that Y lectures fourth and T lectures first. Since this question specifies who lectures first, the last rule is likely to prove crucial: since P does not lecture first, by the contrapositive we must conclude that V lectures before Y:
  • ..... ..... ..... ..... P1 :dbl: V :longline: Y
Since S must be one of the first three professors to lecture, the second and third positions must be assigned to V and S, not necessarily in that order:
oct12_game_2_#11_diagram_1.png
Accordingly, answer choice (C) must be true and is correct. The remaining answer choices do not have to be true.
You do not have the required permissions to view the files attached to this post.

Get the most out of your LSAT Prep Plus subscription.

Analyze and track your performance with our Testing and Analytics Package.